Conversion of cartesian coordinates to polar coordinates

Click For Summary
The discussion focuses on converting a double integral from Cartesian to polar coordinates and confirming the limits of integration. The original integral is straightforward in Cartesian form, but there is uncertainty about the theta limits in polar coordinates after sketching the area. A participant points out that the integral does not represent the area unless the function being integrated is equal to one. This clarification leads to a realization that the area of integration and the function being integrated are distinct concepts. The conversation highlights the importance of understanding these differences in double integrals.
CostasDBD
Messages
2
Reaction score
0
1. Was wondering if anyone could help me confirm the polar limits of integration for the below double integral problem. The question itself is straight forward in cartesian coordinates, but in polar form, I'm a bit suspect of my theta limits after having sketched the it out. any help much appreciated.



2. Homework Equations

\int^{6}_{0}\int^{y}_{0}xdxdy

in polar form:

\int^{\frac{\pi}{2}}_{\frac{\pi}{4}}\int^{6cosec\theta}_{0} r^{2}cos\theta dr d\theta


The Attempt at a Solution



Using a trig substitution over pi/2 and pi/4, i get an answer of 36. it's just when i sketch it, i get a triangle which only has half that area. am i missing something obvious? cheers
 
Physics news on Phys.org
Welcome to PF!

Hi CostasDBD! Welcome to PF! :smile:

(have a pi: π and a theta: θ and an integral: ∫ :wink:)
CostasDBD said:
Using a trig substitution over pi/2 and pi/4, i get an answer of 36. it's just when i sketch it, i get a triangle which only has half that area. am i missing something obvious? cheers

Yup! :redface:

∫∫ x dxdy isn't the area! :wink:
 
Remember, the area of integration and the actual integration are 2 separate entities. Only when the function you're integrating over is f(x,y) = 1 is the integral equal to the area of your area of a double integration.
 
Thanks guys. funny what you pick up when you go back a few pages and have a quick read hey.
this forum is fantastic.
 
Question: A clock's minute hand has length 4 and its hour hand has length 3. What is the distance between the tips at the moment when it is increasing most rapidly?(Putnam Exam Question) Answer: Making assumption that both the hands moves at constant angular velocities, the answer is ## \sqrt{7} .## But don't you think this assumption is somewhat doubtful and wrong?

Similar threads

  • · Replies 19 ·
Replies
19
Views
2K
  • · Replies 5 ·
Replies
5
Views
2K
  • · Replies 11 ·
Replies
11
Views
3K
  • · Replies 3 ·
Replies
3
Views
2K
Replies
7
Views
2K
  • · Replies 22 ·
Replies
22
Views
2K
  • · Replies 6 ·
Replies
6
Views
2K
Replies
3
Views
2K
  • · Replies 14 ·
Replies
14
Views
3K
  • · Replies 3 ·
Replies
3
Views
2K